Difference between revisions of "2022 AMC 12A Problems/Problem 25"
m |
Sugar rush (talk | contribs) |
||
Line 1: | Line 1: | ||
==Problem== | ==Problem== | ||
− | A circle with integer radius <math>r</math> is centered at <math>(r, r)</math>. Distinct line segments of length <math>c_i</math> connect points <math>(0,a_i)</math> to <math>(b_i, 0)</math> for <math>1 \ | + | A circle with integer radius <math>r</math> is centered at <math>(r, r)</math>. Distinct line segments of length <math>c_i</math> connect points <math>(0, a_i)</math> to <math>(b_i, 0)</math> for <math>1 \le i \le 14</math> and are tangent to the circle, where <math>a_i</math>, <math>b_i</math>, and <math>c_i</math> are all positive integers and <math>c_1 \le c_2 \le \cdots \le c_{14}</math>. What is the ratio <math>\frac{c_{14}}{c_1}</math> for the least possible value of <math>r</math>? |
− | <math>c_1 \ | + | |
+ | <math>\textbf{(A)} ~\frac{21}{5} \qquad\textbf{(B)} ~\frac{85}{13} \qquad\textbf{(C)} ~7 \qquad\textbf{(D)} ~\frac{39}{5} \qquad\textbf{(E)} ~17 </math> | ||
==Solution== | ==Solution== | ||
+ | Suppose that with a pair <math>(a_i,b_i)</math> the circle is an excircle. Then notice that the hypotenuse must be <math>(r-x)+(r-y)</math>, so it must be the case that <cmath>a_i^2+b_i^2=(2r-a_i-b_i)^2.</cmath> Similarly, if with a pair <math>(a_i,b_i)</math> the circle is an excircle, the hypotenuse must be <math>(x-r)+(y-r)</math>, leading to the same equation. | ||
− | + | Notice that this equation can be simplified through SFFT to <cmath>(a_i-2r)(b_i-2r)=2r^2.</cmath> Thus, we want the smallest <math>r</math> such that this equation has at least <math>14</math> distinct pairs <math>(a_i,b_i)</math> for which this holds. The obvious choice to check is <math>r=6</math>. In this case, since <math>2r^2=2^3\cdot 3^2</math> has <math>12</math> positive factors, we get <math>12</math> pairs, and we get another two if the factors are <math>-8,-9</math> or vice versa. One can check that for smaller values of <math>r</math>, we do not even get close to <math>14</math> possible pairs. | |
− | |||
− | |||
− | |||
− | |||
− | |||
− | |||
− | |||
− | |||
− | |||
− | |||
− | |||
− | |||
− | |||
− | |||
− | |||
− | |||
− | |||
− | |||
− | |||
− | |||
− | |||
− | |||
− | |||
− | < | ||
− | |||
− | |||
− | |||
− | |||
− | |||
− | |||
− | </cmath> | ||
− | |||
− | |||
− | |||
− | |||
− | |||
− | |||
− | |||
− | |||
− | |||
− | |||
− | |||
− | |||
− | |||
− | |||
− | |||
− | |||
− | |||
− | |||
− | |||
− | |||
− | |||
− | |||
− | |||
− | |||
− | <math> | ||
− | |||
− | |||
− | |||
− | <math> | ||
− | |||
− | |||
− | |||
− | |||
− | |||
− | |||
− | |||
− | |||
− | |||
− | |||
− | |||
− | |||
− | |||
− | + | When <math>r=6</math>, the smallest possible <math>c</math>-value is clearly when the factors are negative. When this occurs, <math>a_i=4, b_i=3</math> (or vice versa), so the mimimal <math>c</math> is <math>5</math>. The largest possible <math>c</math>-value occurs when the largest of <math>a_i</math> and <math>b_i</math> are maximized. This occurs when the factors are <math>72</math> and <math>1</math>, leading to <math>a_i=84, b_i=13</math> (or vice-versa), leading to a maximal <math>c</math> of <math>85</math>. | |
− | + | Hence the answer is <math>\frac{85}5=\boxed{17}</math>. | |
− | + | ~ bluelinfish | |
+ | ==See also== | ||
{{AMC12 box|year=2022|ab=A|num-b=24|after=Last Problem}} | {{AMC12 box|year=2022|ab=A|num-b=24|after=Last Problem}} | ||
{{MAA Notice}} | {{MAA Notice}} |
Revision as of 21:04, 11 November 2022
Problem
A circle with integer radius is centered at . Distinct line segments of length connect points to for and are tangent to the circle, where , , and are all positive integers and . What is the ratio for the least possible value of ?
Solution
Suppose that with a pair the circle is an excircle. Then notice that the hypotenuse must be , so it must be the case that Similarly, if with a pair the circle is an excircle, the hypotenuse must be , leading to the same equation.
Notice that this equation can be simplified through SFFT to Thus, we want the smallest such that this equation has at least distinct pairs for which this holds. The obvious choice to check is . In this case, since has positive factors, we get pairs, and we get another two if the factors are or vice versa. One can check that for smaller values of , we do not even get close to possible pairs.
When , the smallest possible -value is clearly when the factors are negative. When this occurs, (or vice versa), so the mimimal is . The largest possible -value occurs when the largest of and are maximized. This occurs when the factors are and , leading to (or vice-versa), leading to a maximal of .
Hence the answer is .
~ bluelinfish
See also
2022 AMC 12A (Problems • Answer Key • Resources) | |
Preceded by Problem 24 |
Followed by Last Problem |
1 • 2 • 3 • 4 • 5 • 6 • 7 • 8 • 9 • 10 • 11 • 12 • 13 • 14 • 15 • 16 • 17 • 18 • 19 • 20 • 21 • 22 • 23 • 24 • 25 | |
All AMC 12 Problems and Solutions |
The problems on this page are copyrighted by the Mathematical Association of America's American Mathematics Competitions.